LSAT and Law School Admissions Forum

Get expert LSAT preparation and law school admissions advice from PowerScore Test Preparation.

 Administrator
PowerScore Staff
  • PowerScore Staff
  • Posts: 8916
  • Joined: Feb 02, 2011
|
#34864
Complete Question Explanation

Parallel Flaw—SN. The correct answer choice is (C)

Here the author presents one conditional statement and draws a questionable conclusion, which becomes clearer when the statements are diagrammed:
  • Premise: If the theories of the party had been sound, and they had been able to successfully implement those ..... ..... ..... ..... theories, the inflation rate would have decreased:
  • sound theories
    ..... ..... + ..... ..... ..... :arrow: ..... inflation rate decrease
    successful implementation
Contrapositive:
  • ..... ..... ..... ..... ..... ..... ..... sound theories
    inflation rate decrease ..... :arrow: ..... ..... or
    ..... ..... ..... ..... ..... ..... ..... successful implementation
Flawed Conclusion:
  • Since inflation actually increased, the theories must have been unsound:

    inflation rate decrease ..... :arrow: ..... sound theories
The author has not considered the fact that those theories may have been sound (if the implementation had been unsuccessful).

So, the author has presented a conditional rule that leaves two possibilities, but narrows those down to one without justification. The question that follows requires you to find the choice that reflects the same type of flawed conditional reasoning.

Answer choice (A): This choice goes off track in the very first sentence. Unlike the stimulus, which opens with a conditional “and” statement, this choice begins with a conditional “or” statement:

If there had been an invasion, or a climate change, there would have been architectural changes:
  • Invasion
    ..... or ..... ..... :arrow: ..... architectural changes
    climate change
The contrapositive of would be:
  • ..... ..... ..... ..... ..... ..... invasion
    architectural changes ..... :arrow: ..... +
    ..... ..... ..... ..... ..... ..... climate change.
The author then concludes that if there are no architectural changes, there was no invasion:
  • architectural changes ..... :arrow: ..... invasion
This is a valid conclusion, so this choice cannot parallel the flawed reasoning found in the stimulus.

Answer choice (B): This answer choice opens with “many people fear,” which is different from the unqualified conditional statement with which the author begins the stimulus. Many people fear, this choice provides, that if the opposition party wins, and it cuts wages, there will be a strike:
  • opposition party wins
    ..... + ..... ..... ..... :arrow: ..... strike
    party cuts wages
But because the workers have promised not to strike, they must predict that there will be no dramatic wage cut:
  • workers promised not to strike ..... :arrow: ..... workers believe that there won’t be wage cuts
This is flawed reasoning, but it is not analogous to the conditional flaw reflected in the stimulus’ reasoning.

Answer choice (C): This is the correct answer choice. Here, the author presents a conditional statement whose contrapositive points to two possibly necessary conditions:

If the company had been able to sell its subsidiaries, and used cash to buy the new patent, it would have increased in stock price:
  • Sell subsidiaries
    ..... + ..... ..... ..... :arrow: ..... increase stock price
    bought patent with cash
The contrapositive would be as follows:
  • ..... ..... ..... ..... ..... ..... sell subsidiaries
    increased stock price ..... :arrow: ..... ..... or
    ..... ..... ..... ..... ..... ..... used cash to buy the patent
The author then notes that the stock price did not increase, and wrongfully concludes that the company must not have been able to sell its subsidiaries.
  • increased stock price ..... :arrow: ..... sell subsidiaries
Just like the author of the stimulus, this author fails to consider that maybe the company did sell its subsidiaries, but did not buy the patent with cash.

Since this choice perfectly parallels the flawed reasoning in the stimulus, it is confirmed as the right answer choice.

Answer choice (D): In the stimulus, the author presents a conditional “and” statement, and misunderstands the distinction between “and” and “or” of the contrapositive. This answer choice is flawed, but is a kind of modified Mistaken Reversal:

If the battle was won and the rebels were freed, residents were expected to show a lot of support:
  • battle won
    ..... + ..... :arrow: ..... support
    rebels freed
Because residents have shown a lot of support, the author concludes that the battle was won:
  • support ..... :arrow: ..... battle won
Since this conditional flaw does not parallel the conditional flaw found in the stimulus, this choice can be safely ruled out of contention.

Answer choice (E): The stimulus begins with a conditional statement with two sufficient conditions, unlike this choice, which opens with a conditional statement that has two necessary conditions:
  • ..... ..... ..... ..... ..... ..... ..... ..... ..... forecasts more accurate
    new equipment worth the investment ..... :arrow: ..... ..... +
    ..... ..... ..... ..... ..... ..... ..... ..... ..... ratings increase
The author then concludes, based on decreased ratings, that the new equipment provided no improvement over the old. This is somewhat different from being worth the investment, so this conclusion is not justified based on the information provided. The flaw in this case is different from the conditional reasoning flaw reflected in this stimulus, though, so this choice should be eliminated from contention.
 jschruhl
  • Posts: 8
  • Joined: Aug 30, 2012
|
#5902
Hi!

First, thanks so much for answering questions on your forums. Very helpful. In the stimulus of this question, I am confused as to which piece is the sufficient cond. and which is the necessary cond. I am usually pretty decent at identifying these so I am not sure if I am missing something obvious. How is this stimulus diagrammed? It appeared to me that "if the inflation rate had lessened, then the party's economic theories were sound and the program had been successfully implemented," but I believe I may have gotten it backwards. I oftentimes find that flaw questions such as these give me a decent amount of trouble, do you have any tips for what the quickest, best way to solve these are? For example, I believe I reversed the sufficient and necessary conditions in the stimulus which led me to choose the wrong answer of E, but it just didn't seem clear.

Also, is the flaw that they are assuming it has to be because "the theories were far off the mark," when it also could be because the the program was not successfully implemented?

Thanks so much!
 Steve Stein
PowerScore Staff
  • PowerScore Staff
  • Posts: 1153
  • Joined: Apr 11, 2011
|
#5916
Greetings!

First, thanks for your message--it's always good to hear that the forum is helpful. As for the question that you asked about, it's a tough one, in part because we need to understand not just that a piece of reasoning is unsound, but exactly how it's flawed.

In that one, the author opens with a conditional statement:

Had there been sound theories and successful implementation, inflation would have been reduced:

Sound theories
..... and ..... ..... ..... :arrow: lower inflation
Successful implementation



The contrapositive would then be as follows:


..... ..... ..... ..... ..... ..... ..... ..... ..... NOT sound theories
NOT lower inflation ..... :arrow: ..... ..... ..... ..... ..... OR
..... ..... ..... ..... ..... ..... ..... ..... NOT successful implementation

As we can see from the above diagram, higher inflation could mean unsound theories, OR it could also mean unsuccessful implementation.

The author, however, jumps to the unjustified conclusion that the higher inflation had to be attributable to unsound theories.

In correct answer choice C, the author jumps to an analogously flawed conclusion.

Tough one! I hope that's helpful! Let me know whether it's clear--thanks!

~Steve
 jschruhl
  • Posts: 8
  • Joined: Aug 30, 2012
|
#5917
Yes, I definitely see the flaw now. However, I am still unsure as to how you knew which pieces were sufficient and which were necessary. There doesn't seem to be any obvious indicators, and I diagrammed it backwards. How did you know that the economic theories and successful program implementation were sufficient.

I feel like I could read it both ways as an "If, then" statement." Am I missing something?

Thanks!
 Steve Stein
PowerScore Staff
  • PowerScore Staff
  • Posts: 1153
  • Joined: Apr 11, 2011
|
#5925
Thanks for your response. I think you're on to something when you refer to changing it to an "if...then" statement. Take another look at the first sentence, and see where you would put an "if," and where you would put the "then."

Let me know your thoughts--thanks!

~Steve
 ltoulme
  • Posts: 25
  • Joined: Feb 05, 2014
|
#30342
Hi PowerScore,

I see why (C) is correct, but I'm having trouble seeing why (B) is incorrect. Is (B) incorrect because the "workers have promised not to strike" rather than "the workers didn't strike"? Could the workers think that the opposition party isn't going to win the election?

Thanks so much!
Laura
 Adam Tyson
PowerScore Staff
  • PowerScore Staff
  • Posts: 5153
  • Joined: Apr 14, 2011
|
#30405
Hey Itoulme! Answer B looks to have a couple of key differences that keep it from paralleling our stimulus here.

First, looking at Steve's diagram we can see that the stimulus is based on an original conditional claim with two suffient conditions joined by "and" leading to a single necessary condition. The contrapositive gives us one negated sufficient condition leading to two necessary conditions joined by an "or". The author then concludes that that sufficient condition proves that one of the two necessary conditions must occur, when it could be that only the other one does (the nature of "or" being "at least one").

In answer B we have an original claim that looks a lot like that in the stimulus, with the same sort of "and" structure in the sufficient, and the same sort of mistake in the contrapositive. You nailed that - it could be that the other thing happens instead - the opposition doesn't win. So far, so good.

Here's where the problems begin. In our stimulus, the two original sufficient conditions were independent of one another. They can be sound but not succeed in implementing, or succeed in implementing but not be sound, or both, or neither. What about in B? Can the opposition party keep their promise to cut wages if they do not win? They can win and not cut wages, but they cannot cut wages if they do not first win, right? That right there makes this one different on a somewhat fundamental level, in my opinion.

Let's say you don't like that answer because it requires outside info, that only winners can implement policy. Okay, no worries, because there is another issue at work here, and that's fear, promises, and belief. In our stimulus, the situation was purely about facts - what will happen if something else happens. In answer B, though, we aren't dealing with what will happen but instead what some folks fear will happen. Those fears could be unfounded - maybe the workers still would not have gone on strike. Then, in the contrapositive part of the analysis, we don't get info that the workers will not strike, but only that they promised not to do so. Is a promise the same as a fact? Sadly, no - promises can be broken. Then, getting past that, we delve into the likely beliefs of the workers. The stimulus isn't about what they believe, but about what will actually happen. All of these issues in answer B keep it from paralleling our stimulus as well as answer C.

Cool? Keep at it!
 ltoulme
  • Posts: 25
  • Joined: Feb 05, 2014
|
#30419
Hey Adam,

Thanks for your help! I definitely see what you mean about fear/belief in (B) compared to the stimulus. However, I'm still a little unsure about one factor. I obviously see how (B)'s sufficient conditions are not independent of one another; however, I don't see how that doesn't also apply to (C) as a flaw then. Can't they not use the cash unless they sell the subsidiaries? In fact, that's why I put (C) as a loser initially because I didn't think the two sufficient conditions were independent like the stimulus.

Thanks so much for your help!
Laura
User avatar
 Jonathan Evans
PowerScore Staff
  • PowerScore Staff
  • Posts: 726
  • Joined: Jun 09, 2016
|
#30433
Hi, Laura,

Let's simplify this situation somewhat. Before we consider the hypothetical dependence or independence of these conditions on or from one another, which as Adam notes invites outside, extraneous assumptions, flaw questions are best tackled by zeroing in on the flaw itself. Be wary of tangents and digressions that may prove futile.

Describe the basic structure of the argument. Then, use this description as a template for the answer choices. You may symbolize if you wish, but I find it helpful first to have an accurate description of the mechanics and the flaw before I start making notes.

In this case as noted above we have as a premise the conjunction of two propositions (A & B) that implies a third proposition (C). The conclusion (~C :arrow: ~A) is a flawed riff off the contrapositive (~C :arrow: ~A or ~B).

Use this basic description as a point of departure. Be aggressive with process of elimination:

A is out because it's an OR/disjunction operation, evident in the first clause.

B with the "many people fear"/"workers promised" construction just becomes a mess right off the bat.

C gives you a match.

D is a mistaken reversal.

E just doesn't follow the template at all.

Start simple. If you come down to two contenders, then start parsing them out more, but always remember to focus on the flaw first. I can remember a couple situations in which one PtF answer choice was a better match for the elements of the argument in the stimulus and another answer choice was an exact match for the flaw. Guess which one was right: the one that matched the flaw exactly. You have to start with the basic issue and go from there.
 ltoulme
  • Posts: 25
  • Joined: Feb 05, 2014
|
#30504
Thanks very much!

Get the most out of your LSAT Prep Plus subscription.

Analyze and track your performance with our Testing and Analytics Package.